Solving a Frobenius Equation: Finding a Regular Point at x = 0

Click For Summary
The discussion focuses on solving a Frobenius equation to show that the differential equation has a regular point at x=0. The equation is rearranged to identify x=0 as a singular point, leading to the need for coefficients in the series solution. The participants emphasize the importance of selecting the correct form for the series and choosing the constant c to ensure a0 is not zero. They clarify that the coefficients for higher-order terms depend on either a0 or a1, highlighting the challenge of using standard power series due to the nature of the singular point. The conversation underscores the necessity of applying the Frobenius method effectively to find the solution.
01jbell
Messages
3
Reaction score
0
hey i am stuck on this question for my ode course its using frobunius

4. show that the equation

yii + 1/x yi + (1-1/(4*x^2))y = 0

has a regual point at x=0
using the method of frobenius assuming a solution of the form

y=\sum ar xc+r

show that the idical equation is c^2=1/4


thanks for nay help given
 
Physics news on Phys.org
What have you managed so far?
 
i have proved that x=0 is a singular point and i rearanged the ode to get

x^2 yii + x y^i + x^2 y -1/4 y =0

however trying to work out coeff of xr , xr+1 etc etc is the probelm because i get

a0*(r^2-0.25) = 0
a1*{(r+1.5)*(r+0.5)}=0

and i thought u would have ot get a a0 in the equation for a1
 
The r's in your equations should be c's, but otherwise they look okay.

If you solve for the coefficients for r≥2, you'll see they depend on the either a0 or a1.
 
Because this 0 is a "regular singular point" for this problem, you cannot use the standard power series. You will have to use "Frobenious' method"- try something of the form
y= \sum_n a_nx^{n+c}

Choose c so that a0 is NOT 0.
 
Question: A clock's minute hand has length 4 and its hour hand has length 3. What is the distance between the tips at the moment when it is increasing most rapidly?(Putnam Exam Question) Answer: Making assumption that both the hands moves at constant angular velocities, the answer is ## \sqrt{7} .## But don't you think this assumption is somewhat doubtful and wrong?

Similar threads

  • · Replies 4 ·
Replies
4
Views
3K
Replies
8
Views
2K
  • · Replies 2 ·
Replies
2
Views
915
  • · Replies 2 ·
Replies
2
Views
5K
  • · Replies 3 ·
Replies
3
Views
1K
  • · Replies 4 ·
Replies
4
Views
3K
  • · Replies 1 ·
Replies
1
Views
2K
  • · Replies 1 ·
Replies
1
Views
2K
  • · Replies 4 ·
Replies
4
Views
3K
  • · Replies 27 ·
Replies
27
Views
2K